ML15313A063: Difference between revisions

From kanterella
Jump to navigation Jump to search
(Created page by program invented by StriderTol)
(Created page by program invented by StriderTol)
 
Line 16: Line 16:


=Text=
=Text=
{{#Wiki_filter:ES-401 W ritten Exam i nation Review W orksheet C LIN TO N 2015 Form ES-401-9 1. 2. 3. Psychometric Flaws 4. Job Content Flaws 5. Other 6. 7. 8. Q# LOK LOO (F/H) (1-5) Stem Cues T/F Cred. Partial Job-Minutia #I Back-Q= SRO B/M/N U/E/S Explanation Focus Dist. Link units ward KIA Only 1 L 3 x x y N u The stem states " IRM detectors are withdrawn from the core region during power operation within a tube which extends from the core p l ate up to the top guide." This question has two problems 1) the IRMs run i n dry tubes that extend from the be l ow the reactor vessel to the top gu i de. So there is no correct answer. 2) the dry tub is inserted into a guide tub . So both the " IRMs are within a dry tube" and " the IRMs are with in a guide tube" would be correct. RESOLUTION
{{#Wiki_filter:ES-401                                     Written Examination Review Worksheet CLINTON 2015                                                         Form ES-401-9
: Question replaced 2 H 3 y B s 3 L 3 y B E The stem now reads ''Which ONE of the following will generate a reactor scram SIGNAL as a result of this loss of power?" Consider changing " SIGNAL* to " input signal". The stem condit ions would o nly be a 'h scram? RESOLUTION
: 1. 2.       3. Psychometric Flaws         4 . Job Content Flaws     5. Other   6. 7.                                     8.
: Change made Instructions (Refer to Section D of ES-401 and Appendix B for additional information regarding each of the following concepts.) 1. En t er the level of knowledge (LOK) of each question as either (F)undamental or (H)igher cognitive level. 2. En t er the level of difficulty (LOO) of each question using a 1 -5 (easy -difficult) rating scale (questions i n the 2 -4 range are acceptable). 3. Check the appropriate box if a psychometric flaw is identified
Q# LOK   LOO (F/H) (1-5) Stem Cues     T/F   Cred. Partial Job- Minutia #I       Back- Q=   SRO B/M/N U/E/S                               Explanation Focus               Dist.       Link           units ward   KIA   Only 1   L     3     x                         x                                 y         N     u   The stem states "IRM detectors are withdrawn from the core region during power operation within a               tube which extends from the core plate up to the top guide." This question has two problems
: The stem lacks sufficient focus to elicit the correct answer (e.g., unclear in tent , more information is needed , or too much needless information).
: 1) the IRMs run in dry tubes that extend from the below the reactor vessel to the top guide. So there is no correct answer.
The stem or distractors contain cues (i.e., clues. specific determiners , phrasing , length , etc). The answer choices are a collection of unrelated true/false statements. The distractors are not credible; single implausib le distractors should be repaired , more than one is unacceptable. One or more distractors is (are) partially correct (e.g., i f the applicant can make unstated assumptions that are not contradicted by stem). 4. Check the appropriate box if a job content error is identified
: 2) the dry tub is inserted into a guide tub . So both the "IRMs are within a dry tube" and "the IRMs are with in a guide tube" would be correct.
: The question is not linked to t he job requirements (i.e., the question has a valid KIA but , as written , is not operational in con tent). The question requires the recall of knowledge that is too specific for the closed reference test mode (i.e., it is not required to be known from memory). The question contains data with an unrealistic leve l of accuracy or inconsis tent units (e.g., panel meter in percent with question in gallons). The question requires reverse logic or application compared to the job requ irements. 5. Check questions that are sampled fo r conformance with the approved KIA and those that are designated SRO-only (KIA and license level mismatches are unacceptable). 6. Enter question source: (B)ank , (M)odified , or (N)ew. Check that (M)odified questions meet criteria of ES-401 Section 0.2.f. 7. Based on the reviewer's judgment , is the question as written (U)nsatis factory (requiring repair or replacement), in need of (E)di toria l enhancement , or (S)atisfactory?
RESOLUTION: Question replaced 2   H     3                                                                 y         B     s 3   L     3                                                                 y         B     E   The stem now reads ''Which ONE of the following will generate a reactor scram SIGNAL as a result of this loss of power?" Consider changing "SIGNAL* to "input signal". The stem conditions would only be a 'h scram?
: a. At a minimum , explain any " U" ratings (e.g., how the Append ix B psychometric attributes are not being met).
RESOLUTION: Change made
ES-401 2 F o rm ES-4 0 1-9 1. 2. 3. Psychometric Flaws 4. Job Content Flaws 5. Other 6. 7. 8. Q# LOK L OO (FIH) (1-5) Stem Cues T/F Cred. Partial Job-Minutia #I Back-Q= SRO B/M/N U/E/S Explanation Focus Dist Link units ward KIA Only 4 H 3 y B s 5 H 4 y B s 6 H 4 y N s 7 H 2 y N s 8 L 4 y B s 9 L 3 y N s 10 H 4 y 8 s 11 H 3 y N s 12 H 3 y B s 13 H 2 y N u This quest ions has 2 problems: 1) If in the condition of the stern the Governor is held in the raise posi t ion long enough to see a r i se in both watts and current I believe you w o u ld a l so see a rise in vars. Then there wou l d be n o co rr ect answer. (on t he simu l ator try raising load from 3000 to 3500 Kw and see if vars change.) 2) JPM " Parallel DG 18 With Offsite Power* will dup lic ate th i s question. RESO L UTIO N: Quest ion revised 1 4 H 2 y N s 15 H 3 y B s 16 H 4 y N s 17 L 3 y B s 18 H 3 y N s
 
: 1. 2. 3. PsychOmetric Flaws 4. Job Co nt ent Flaws 5. O t he r 6. 7. 8. Q# LO K LOO (F I H) (1-5) Stem Cues T/F Cred Partial Job-M inuti a #I Back-Q= SRO B/M/N U/E/S Exp l anation Focus Dist. Link units ward KJA Only 1 9 H 3 y B s 20 H 3 y B s 21 H 3 x y N IJ I do not think this is an RO leve l TS question as written. Conside r E changing the bullets to -1 E21-F011 LP CS Pump Min F low Rec ir c Valve be closed with fuses removed. -AUTO start of the LPCS disab le d. RESOLUTION
Instructions (Refer to Section D of ES-401 and Appendix B for additional information regarding each of the following concepts.)
: Site s u pported that the question met the expectat i ons o f ROs a t C l inton . Editorial ch a nges made. 22 H 3 y B s 23 H 2 y N s 2 4 L 3 x y B u Between LPR M calibrat i ons APRM gains would be adjusted to accou nt for LPRM detector U235 dep le tion. RESOLUTIO N: Th e second correct a n swer was changed. 25 H 3 y B s 26 L 3 y N s 27 L 3 x y M u If " D" is co r rect then " a** will be correct. Consider changing " A" and " B" (1) to 'primary b ut NOT secondary" . RESOLUTIO N: Requested change made 28 l 3 y B s 29 l 3 y B s 30 L 3 y B s 31 H 3 y N s 32 L 3 y N s 33 L 4 y N s 34 H 3 y N s
: 1. Enter the level of knowledge (LOK) of each question as either (F)undamental or (H)igher cognitive level.
: 1. 2. 3. Psychometric Flaws 4. Jo b Content Flaws 5. O the r 6. 7. 8. Q# LO K LOO (F/H) (1-5) Stem C u es T/F Cred. Partia l Jo b-M in utia #I Back-Q= SRO B/M/N U/E/S Explanation Focus D ist. Link units ward KI A Only 35 H 2 y B s 36 L 3 y N s 37 l 3 x y B y " B" p revent b o r on dil ut ion , *c* p re ven t injection inside the shroud are E both correct. See EO P-1A r eference provided.
: 2. Enter the level of difficulty (LOO) of each question using a 1 - 5 (easy - difficult) rating scale (questions in the 2 - 4 range are acceptable).
RESOLUTION:
: 3. Check the appropriate box if a psychometric flaw is identified:
The stem was c han ged to c la rify th e ques t ion. 38 L 2 y N s 39 H 3 y N s 40 L 3 y 6 s 41 l 3 y B s 42 H 3 y M s 43 H 3 y B s 44 l 3 y N s 4 5 L 2 y B E List the distracto rs lowest t o highest. i.e. switch " B' and " C". RESO LU TIO N: Change made 4 6 H 3 y N s 47 H 3 y N s 48 L 3 y B s 4 9 H 3 x y B E 1) For th e conditio ns g i ven i n the stem is i t expec ted that the cool down rate w ill inc rease o r de crease with time? *s* will be correct on l y with an increas ing o r constant cool down rate. 2) For distrac te r " A' should it be changed to " Shift RCIR discharge to the reactor vesse l"? Because with RCIR in tank to tank vessel leve l would be unchanged by raising flow. RESOLUTIO N: Stem and distracter changed 50 H 2 y B s
The stem lacks sufficient focus to elicit the correct answer (e.g ., unclear intent, more information is needed, or too much needless information).
: 1. 2. 3. Psychometric Flaws 4. Job Content Flaws 5. Other 6. 7. 8. Q# LOK LOO (F/H) (1-5) Stem Cues T/F Cred. Partial Job-Minut ia #I Back-Q= SRO B/M/N U/E/S Explanation Focus Dist. Link units ward KIA Only 51 L 3 x y N u It is not clear in the stem if in part (2) of the question we are asking for the action to be taken at the t ime level is 18ft 9inches (answer D), or at t he 15ft 1 inch (Answer C) RESOLUTION
The stem or distractors contain cues (i.e., clues. specific determiners, phrasing, length, etc).
: Question revised. 52 L 3 y B E Change " B" to , " lowest possible driving head and flow fr o m the primary system!:tO reduce the discharge rate." RESOLUTION
The answer choices are a collection of unrelated true/false statements.
: Change made 53 L 3 y B s 54 H 3 y M s 55 L 3 y N s 56 H 3 y N s 57 H 4 y N E I th ink we need to provide the candidates with a time after the loss of offsite power that the plant conditions are current for. (th e fact that an SRV is open after a LOFP doesn't= ATWS at time 0) Also do you think it is fair to ask an operator to diagnose an A TWS and not give them control rod or APRM info. Maybe we should say you are the BOP and observe the conditions. RESOLUTION
The distractors are not credible; single implausible distractors should be repaired , more than one is unacceptable.
: Added time = 5 minutes. And justified question based on operator training.
One or more distractors is (are) partially correct (e.g., if the applicant can make unstated assumptions that are not contradicted by stem).
58 H 3 y M s 59 H 3 y B s 60 H 3 x y B l:J For the conditions of the stem I do not believe you can " main ta in E steady states power and balance loop flows" to maintain steady states power for the stem conditions it will require an imbalance in loop flow. RESOLUTION
: 4. Check the appropriate box if a job content error is identified:
: The conditions of the stem did not require a flow change to comply with Tec h Specs , therefore only editorial changes needed to be made 61 L 3 y B s
The question is not linked to the job requirements (i.e., the question has a valid KIA but, as written, is not operational in content).
: 1. 2. 3. Psychome tric Fla w s 4. Job Content Flaws 5. Other 6. 7. 8. Q# LO K LOO (F/H) (1-5) Stem Cues T/F Cred. Partial Job-Minutia #I Back-Q= SRO B/M/N U/E/S Exp lan at i on Focus Dist. Link un i t s ward KIA Only 62 L 4 y B s Comment: add an explanation for " D" being incorrect t o the answer key. RESOLUT I O N: Done 63 H 3 y B s 64 L 4 y B s 65 H 4 y B s 66 H 3 y N s 67 L 3 y N s 68 L 3 y B s 69 L 2 y N s 70 L 3 y B s 71 L 2 y B s 72 H 3 y N s 73 H 3 x y B y For a rod drop as descr i bed in t he stem there wo uld be a ROD s DRIFT A larm. Therefore  
The question requires the recall of knowledge that is too specific for the closed reference test mode (i.e., it is not required to be known from memory).
" A" wou ld be wrong for the same re ason as M C''. RESOLUTION
The question contains data with an unrealistic level of accuracy or inconsistent units (e.g., panel meter in percent with question in gallons).
: Site explained why t he r e would be n o ROD DR IF T A l arm. 74 H 4 x y N y CPS 401 0.01 step 4.2. 4. Specifically call out Condenser Vacuum E Pumps as needing to be considered for ra d condit ion s. Therefore  
The question requires reverse logic or application compared to the job requirements.
" C" is correct. Ed i torial: change S t em to. Under these conditio ns , when 12erforming the SUBSEQUENT A CTIO N s OF CPS 4100.01, what special precaution must be taken to minimize the radiological impact of this event when ... RESOLUTIO N: Site explained why " C" was i ncorrect.
: 5. Check questions that are sampled fo r conformance with the approved KIA and those that are designated SRO-only (KIA and license level mismatches are unacceptable).
On ly editor i al changes were needed. 75 L 4 y B s
: 6. Enter question source: (B)ank, (M)odified, or (N)ew. Check that (M)odified questions meet criteria of ES-401 Section 0 .2.f.
: 1. 2. 3. Psychometric Flaws 4. Job Content Flaws 5. Other 6. 7. 8. Q# LOK LOO (F/H) (1-5) Stem Cues T/F Cred. Partial Job-M in utia #I Back-Q= SRO B/M/N U/E/S E xplanat ion Focus D ist. Link units wa r d KIA Only s R 0 0 N L y Q u E s T I 0 N s 1 H 3 y y N s 2 H 3 y y N s 3 H 3 y y M s 4 H 3 y y M s 5 H 3 y y N s 6 H 3 x y y N y 1} The question " If the appropr i ate actions are taken , can a E plant startup commence?" is too vague you can always start up after all appropriate action are take n. Did you mean " Does Tech Spec 3.3.1.1 a llo w re start without repair of the failed channels?" 2) Could CH A and D have t ripp ed at 19:42: 12: 15.!1,? I f so cou ld t h is confuse the candidates. RESOLUT I ON: Only editorial changes were needed in this question 7 H 3 y N N y This question is written at RO level. i.e. With current conditions s should feedwater be shutoff, s ho uld HPCI be shutoff , should MSIVs be closed. RESOLUTION
: 7. Based on the reviewer's judgment, is the question as written (U)nsatisfactory (requiring repair or replacement) , in need of (E)ditorial enhancement, or (S)atisfactory?
: Site was able to justify the question as SRO only based on the NRC guide. 8 H 2 y y N s 9 H 3 y N B y This question does not incl ude *selection o f procedures" only RO E level evaluation of plant condi t ion. Can you change to " enter CPS XXX to spray t he containment , etc" RESOLUTION
: a. At a minimum, explain any "U" ratings (e.g., how the Appendix B psychometric attributes are not being met).
: Only editorial changes w here n eeded. 10 H 3 y N B s 11 H 3 y y N s
 
: 1. 2. 3. Psychometric Fla ws 4. Job Content Flaws 5. Other 6. 7. 8. Q# LOK LOO (F/H) (1-5) Stem Cues T/F C r ed. Partial Job-M inutia #I Back-Q= SRO B/M/N U/E/S Explanation F ocus Dist. Li nk units ward KIA Only 12 H 4 y y N s 13 H 3 y y N s 14 H 3 y y N s N ote: this was accepted as SRO because the candi dat e was effectively asked to select a sec ti on of the S B O p r ocedu re to direct. 15 H 3 y y N s 16 H 3 y N N y Th is question can be answered knowing the power source of the s i solation valves and that the valves should function at 1 42 F , RO knowledge. Ma ybe we could ask " Under these conditions sho uld an emergence event FU1be declared." As the seco n d part o f the quest i on. RESOLUTION
ES-401                                                                           2                                                                       Form ES-401-9
: Si te was able to justify the question as SRO only based on the N RC guid e. 17 L 2 y y N s 18 H 3 y y N s 19 H 4 y y B s 20 H 4 y y N s 21 H 3 y y M s 22 H 3 y y B s 23 L 2 y y N s 24 H 2 x y y B u Why would anyone bel ieve recording a potential LCO would preven t a re l ease? Answers " C & D" RESOLUTION
: 1. 2.       3. Psychometric Flaws         4 . Job Content Flaws     5. Other   6. 7.                                         8.
: the question was rewritten. 25 L 3 y y N s ES-401 W ritten Exam i nation Review W orksheet C LIN TO N 2015 Form ES-401-9 1. 2. 3. Psychometric Flaws 4. Job Content Flaws 5. Other 6. 7. 8. Q# LOK LOO (F/H) (1-5) Stem Cues T/F Cred. Partial Job-Minutia #I Back-Q= SRO B/M/N U/E/S Explanation Focus Dist. Link units ward KIA Only 1 L 3 x x y N u The stem states " IRM detectors are withdrawn from the core region during power operation within a tube which extends from the core p l ate up to the top guide." This question has two problems 1) the IRMs run i n dry tubes that extend from the be l ow the reactor vessel to the top gu i de. So there is no correct answer. 2) the dry tub is inserted into a guide tub . So both the " IRMs are within a dry tube" and " the IRMs are with in a guide tube" would be correct. RESOLUTION
Q#   LOK LOO (FIH) (1-5) Stem Cues   T/F   Cred. Partial Job- Minutia #I       Back- Q=   SRO B/M/N U/E/S                                 Explanation Focus               Dist         Link           units ward   KIA Only 4   H     3                                                                 y         B     s 5   H     4                                                                 y         B     s 6   H     4                                                                 y         N     s 7   H     2                                                                 y         N     s 8   L     4                                                                 y         B     s 9     L   3                                                                 y         N     s 10   H     4                                                                 y         8     s 11   H     3                                                                 y         N     s 12   H     3                                                                 y         B     s 13   H     2                                                                 y         N     u   This questions has 2 problems:
: Question replaced 2 H 3 y B s 3 L 3 y B E The stem now reads ''Which ONE of the following will generate a reactor scram SIGNAL as a result of this loss of power?" Consider changing " SIGNAL* to " input signal". The stem condit ions would o nly be a 'h scram? RESOLUTION
: 1) If in the condition of the stern the Governor is held in the raise position long enough to see a rise in both watts and current I believe you would also see a rise in vars. Then there wou ld be no correct answer. (on the simu lator try raising load from 3000 to 3500 Kw and see if vars change .)
: Change made Instructions (Refer to Section D of ES-401 and Appendix B for additional information regarding each of the following concepts.) 1. En t er the level of knowledge (LOK) of each question as either (F)undamental or (H)igher cognitive level. 2. En t er the level of difficulty (LOO) of each question using a 1 -5 (easy -difficult) rating scale (questions i n the 2 -4 range are acceptable). 3. Check the appropriate box if a psychometric flaw is identified
: 2) JPM " Parallel DG 18 With Offsite Power
: The stem lacks sufficient focus to elicit the correct answer (e.g., unclear in tent , more information is needed , or too much needless information).
* will duplicate th is question.
The stem or distractors contain cues (i.e., clues. specific determiners , phrasing , length , etc). The answer choices are a collection of unrelated true/false statements. The distractors are not credible; single implausib le distractors should be repaired , more than one is unacceptable. One or more distractors is (are) partially correct (e.g., i f the applicant can make unstated assumptions that are not contradicted by stem). 4. Check the appropriate box if a job content error is identified
RESOLUTION: Question revised 14    H     2                                                                 y         N     s 15     H     3                                                               y         B     s 16     H   4                                                                 y         N     s 17     L     3                                                               y         B     s 18     H     3                                                               y         N     s
: The question is not linked to t he job requirements (i.e., the question has a valid KIA but , as written , is not operational in con tent). The question requires the recall of knowledge that is too specific for the closed reference test mode (i.e., it is not required to be known from memory). The question contains data with an unrealistic leve l of accuracy or inconsis tent units (e.g., panel meter in percent with question in gallons). The question requires reverse logic or application compared to the job requ irements. 5. Check questions that are sampled fo r conformance with the approved KIA and those that are designated SRO-only (KIA and license level mismatches are unacceptable). 6. Enter question source: (B)ank , (M)odified , or (N)ew. Check that (M)odified questions meet criteria of ES-401 Section 0.2.f. 7. Based on the reviewer's judgment , is the question as written (U)nsatis factory (requiring repair or replacement), in need of (E)di toria l enhancement , or (S)atisfactory?
: 1. 2.       3. PsychOmetric Flaws           4. Job Content Flaws   5. Other 6. 7.                                         8.
: a. At a minimum , explain any " U" ratings (e.g., how the Append ix B psychometric attributes are not being met).
Q# LOK  LOO (FIH) (1-5) Stem Cues   T/F   Cred   Partial Job- Minutia #I Back-   Q= SRO B/M/N U/E/S                                   Explanation Focus             Dist.         Link         units ward KJA Only 19    H     3                                                               y       B   s 20   H     3                                                               y       B   s 21   H     3     x                                                         y       N   IJ I do not think this is an RO level TS question as written. Consider changing the bullets to E
ES-401 2 F o rm ES-4 0 1-9 1. 2. 3. Psychometric Flaws 4. Job Content Flaws 5. Other 6. 7. 8. Q# LOK L OO (FIH) (1-5) Stem Cues T/F Cred. Partial Job-Minutia #I Back-Q= SRO B/M/N U/E/S Explanation Focus Dist Link units ward KIA Only 4 H 3 y B s 5 H 4 y B s 6 H 4 y N s 7 H 2 y N s 8 L 4 y B s 9 L 3 y N s 10 H 4 y 8 s 11 H 3 y N s 12 H 3 y B s 13 H 2 y N u This quest ions has 2 problems: 1) If in the condition of the stern the Governor is held in the raise posi t ion long enough to see a r i se in both watts and current I believe you w o u ld a l so see a rise in vars. Then there wou l d be n o co rr ect answer. (on t he simu l ator try raising load from 3000 to 3500 Kw and see if vars change.) 2) JPM " Parallel DG 18 With Offsite Power* will dup lic ate th i s question. RESO L UTIO N: Quest ion revised 1 4 H 2 y N s 15 H 3 y B s 16 H 4 y N s 17 L 3 y B s 18 H 3 y N s
                                                                                                    -     1 E21-F011 LPCS Pump Min Flow Recirc Valve be closed with fuses removed .
: 1. 2. 3. PsychOmetric Flaws 4. Job Co nt ent Flaws 5. O t he r 6. 7. 8. Q# LO K LOO (F I H) (1-5) Stem Cues T/F Cred Partial Job-M inuti a #I Back-Q= SRO B/M/N U/E/S Exp l anation Focus Dist. Link units ward KJA Only 1 9 H 3 y B s 20 H 3 y B s 21 H 3 x y N IJ I do not think this is an RO leve l TS question as written. Conside r E changing the bullets to -1 E21-F011 LP CS Pump Min F low Rec ir c Valve be closed with fuses removed. -AUTO start of the LPCS disab le d. RESOLUTION
                                                                                                    -     AUTO start of the LPCS disabled .
: Site s u pported that the question met the expectat i ons o f ROs a t C l inton . Editorial ch a nges made. 22 H 3 y B s 23 H 2 y N s 2 4 L 3 x y B u Between LPR M calibrat i ons APRM gains would be adjusted to accou nt for LPRM detector U235 dep le tion. RESOLUTIO N: Th e second correct a n swer was changed. 25 H 3 y B s 26 L 3 y N s 27 L 3 x y M u If " D" is co r rect then " a** will be correct. Consider changing " A" and " B" (1) to 'primary b ut NOT secondary" . RESOLUTIO N: Requested change made 28 l 3 y B s 29 l 3 y B s 30 L 3 y B s 31 H 3 y N s 32 L 3 y N s 33 L 4 y N s 34 H 3 y N s
RESOLUTION: Site supported that the question met the expectations of ROs at Clinton . Editorial changes made.
: 1. 2. 3. Psychometric Flaws 4. Jo b Content Flaws 5. O the r 6. 7. 8. Q# LO K LOO (F/H) (1-5) Stem C u es T/F Cred. Partia l Jo b-M in utia #I Back-Q= SRO B/M/N U/E/S Explanation Focus D ist. Link units ward KI A Only 35 H 2 y B s 36 L 3 y N s 37 l 3 x y B y " B" p revent b o r on dil ut ion , *c* p re ven t injection inside the shroud are E both correct. See EO P-1A r eference provided.
22   H     3                                                               y       B   s 23   H     2                                                               y       N   s 24    L   3                               x                               y       B   u   Between LPRM calibrations APRM gains would be adjusted to account for LPRM detector U235 depletion.
RESOLUTION:
RESOLUTION: The second correct answer was changed.
The stem was c han ged to c la rify th e ques t ion. 38 L 2 y N s 39 H 3 y N s 40 L 3 y 6 s 41 l 3 y B s 42 H 3 y M s 43 H 3 y B s 44 l 3 y N s 4 5 L 2 y B E List the distracto rs lowest t o highest. i.e. switch " B' and " C". RESO LU TIO N: Change made 4 6 H 3 y N s 47 H 3 y N s 48 L 3 y B s 4 9 H 3 x y B E 1) For th e conditio ns g i ven i n the stem is i t expec ted that the cool down rate w ill inc rease o r de crease with time? *s* will be correct on l y with an increas ing o r constant cool down rate. 2) For distrac te r " A' should it be changed to " Shift RCIR discharge to the reactor vesse l"? Because with RCIR in tank to tank vessel leve l would be unchanged by raising flow. RESOLUTIO N: Stem and distracter changed 50 H 2 y B s
25   H     3                                                             y       B   s 26     L     3                                                             y       N   s 27     L     3                             x                               y       M   u   If "D" is correct then "a** will be correct. Consider changing "A" and "B" (1) to ' primary but NOT secondary" .
: 1. 2. 3. Psychometric Flaws 4. Job Content Flaws 5. Other 6. 7. 8. Q# LOK LOO (F/H) (1-5) Stem Cues T/F Cred. Partial Job-Minut ia #I Back-Q= SRO B/M/N U/E/S Explanation Focus Dist. Link units ward KIA Only 51 L 3 x y N u It is not clear in the stem if in part (2) of the question we are asking for the action to be taken at the t ime level is 18ft 9inches (answer D), or at t he 15ft 1 inch (Answer C) RESOLUTION
RESOLUTION: Requested change made 28     l     3                                                             y       B   s 29     l     3                                                             y       B   s 30     L     3                                                             y       B   s 31     H     3                                                             y       N   s 32     L     3                                                             y       N   s 33   L     4                                                             y       N   s 34   H     3                                                             y       N   s
: Question revised. 52 L 3 y B E Change " B" to , " lowest possible driving head and flow fr o m the primary system!:tO reduce the discharge rate." RESOLUTION
: 1. 2.       3. Psychometric Flaws         4. Job Content Flaws     5. Other  6. 7.                                         8.
: Change made 53 L 3 y B s 54 H 3 y M s 55 L 3 y N s 56 H 3 y N s 57 H 4 y N E I th ink we need to provide the candidates with a time after the loss of offsite power that the plant conditions are current for. (th e fact that an SRV is open after a LOFP doesn't= ATWS at time 0) Also do you think it is fair to ask an operator to diagnose an A TWS and not give them control rod or APRM info. Maybe we should say you are the BOP and observe the conditions. RESOLUTION
Q# LOK  LOO (F/H) (1-5) Stem Cues T/F     Cred. Partial Job- Minutia #I     Back- Q=   SRO B/M/N U/E/S                                 Explanation Focus             Dist.         Link         units ward   KIA  Only 35   H     2                                                               y         B     s 36   L     3                                                               y         N     s 37   l     3                             x                                 y         B     y   "B" p revent boron dilution,   *c* prevent injection inside the shroud are both correct. See EOP-1A reference provided.
: Added time = 5 minutes. And justified question based on operator training.
E RESOLUTION: The stem was changed to clarify the quest ion.
58 H 3 y M s 59 H 3 y B s 60 H 3 x y B l:J For the conditions of the stem I do not believe you can " main ta in E steady states power and balance loop flows" to maintain steady states power for the stem conditions it will require an imbalance in loop flow. RESOLUTION
38   L     2                                                               y         N     s 39   H     3                                                               y         N     s 40   L     3                                                               y         6     s 41   l     3                                                               y         B     s 42   H     3                                                               y         M     s 43   H     3                                                               y         B     s 44   l     3                                                               y         N     s 45    L   2                                                               y         B     E   List the distractors lowest to highest. i.e. switch "B' and "C".
: The conditions of the stem did not require a flow change to comply with Tec h Specs , therefore only editorial changes needed to be made 61 L 3 y B s
RESOLUTION: Change made 46  H     3                                                               y         N     s 47   H     3                                                               y         N     s 48   L   3                                                               y         B     s 49  H     3                       x                                       y         B     E         1)   For the conditions given in the stem is it expected that the cool down rate will increase or decrease with time? *s* will be correct only with an increasing or constant cool down rate.
: 1. 2. 3. Psychome tric Fla w s 4. Job Content Flaws 5. Other 6. 7. 8. Q# LO K LOO (F/H) (1-5) Stem Cues T/F Cred. Partial Job-Minutia #I Back-Q= SRO B/M/N U/E/S Exp lan at i on Focus Dist. Link un i t s ward KIA Only 62 L 4 y B s Comment: add an explanation for " D" being incorrect t o the answer key. RESOLUT I O N: Done 63 H 3 y B s 64 L 4 y B s 65 H 4 y B s 66 H 3 y N s 67 L 3 y N s 68 L 3 y B s 69 L 2 y N s 70 L 3 y B s 71 L 2 y B s 72 H 3 y N s 73 H 3 x y B y For a rod drop as descr i bed in t he stem there wo uld be a ROD s DRIFT A larm. Therefore  
: 2)   For distracte r "A ' should it be changed to "Shift RCIR discharge to the reactor vessel"? Because with RCIR in tank to tank vessel level would be unchanged by raising flow.
" A" wou ld be wrong for the same re ason as M C''. RESOLUTION
RESOLUTION: Stem and distracter changed 50   H     2                                                               y         B     s
: Site explained why t he r e would be n o ROD DR IF T A l arm. 74 H 4 x y N y CPS 401 0.01 step 4.2. 4. Specifically call out Condenser Vacuum E Pumps as needing to be considered for ra d condit ion s. Therefore  
: 1. 2.       3. Psychometric Flaws         4. Job Content Flaws   5. Other 6. 7.                                         8.
" C" is correct. Ed i torial: change S t em to. Under these conditio ns , when 12erforming the SUBSEQUENT A CTIO N s OF CPS 4100.01, what special precaution must be taken to minimize the radiological impact of this event when ... RESOLUTIO N: Site explained why " C" was i ncorrect.
Q#   LOK LOO (F/H) (1-5) Stem Cues T/F     Cred. Partial Job- Minutia #I Back-   Q= SRO B/M/N U/E/S                                 Explanation Focus             Dist.         Link         units ward KIA Only 51   L     3     x                                                       y       N   u   It is not clear in the stem if in part (2) of the question we are asking for the action to be taken at the time level is 18ft 9inches (answer D),
On ly editor i al changes were needed. 75 L 4 y B s
or at the 15ft 1 inch (Answer C)
: 1. 2. 3. Psychometric Flaws 4. Job Content Flaws 5. Other 6. 7. 8. Q# LOK LOO (F/H) (1-5) Stem Cues T/F Cred. Partial Job-M in utia #I Back-Q= SRO B/M/N U/E/S E xplanat ion Focus D ist. Link units wa r d KIA Only s R 0 0 N L y Q u E s T I 0 N s 1 H 3 y y N s 2 H 3 y y N s 3 H 3 y y M s 4 H 3 y y M s 5 H 3 y y N s 6 H 3 x y y N y 1} The question " If the appropr i ate actions are taken , can a E plant startup commence?" is too vague you can always start up after all appropriate action are take n. Did you mean " Does Tech Spec 3.3.1.1 a llo w re start without repair of the failed channels?" 2) Could CH A and D have t ripp ed at 19:42: 12: 15.!1,? I f so cou ld t h is confuse the candidates. RESOLUT I ON: Only editorial changes were needed in this question 7 H 3 y N N y This question is written at RO level. i.e. With current conditions s should feedwater be shutoff, s ho uld HPCI be shutoff , should MSIVs be closed. RESOLUTION
RESOLUTION: Question revised.
: Site was able to justify the question as SRO only based on the NRC guide. 8 H 2 y y N s 9 H 3 y N B y This question does not incl ude *selection o f procedures" only RO E level evaluation of plant condi t ion. Can you change to " enter CPS XXX to spray t he containment , etc" RESOLUTION
52   L     3                                                             y       B   E   Change "B" to, "lowest possible driving head and flow from the primary system!:tO reduce the discharge rate."
: Only editorial changes w here n eeded. 10 H 3 y N B s 11 H 3 y y N s
RESOLUTION: Change made 53   L     3                                                             y       B   s 54   H     3                                                             y       M   s 55     L     3                                                             y       N   s 56   H     3                                                             y       N   s 57   H     4                                                             y       N   E   I th ink we need to provide the candidates with a time after the loss of offsite power that the plant conditions are current for. (the fact that an SRV is open after a LOFP doesn't= ATWS at time 0) Also do you think it is fair to ask an operator to diagnose an ATWS and not give them control rod or APRM info. Maybe we should say you are the BOP and observe the conditions.
: 1. 2. 3. Psychometric Fla ws 4. Job Content Flaws 5. Other 6. 7. 8. Q# LOK LOO (F/H) (1-5) Stem Cues T/F C r ed. Partial Job-M inutia #I Back-Q= SRO B/M/N U/E/S Explanation F ocus Dist. Li nk units ward KIA Only 12 H 4 y y N s 13 H 3 y y N s 14 H 3 y y N s N ote: this was accepted as SRO because the candi dat e was effectively asked to select a sec ti on of the S B O p r ocedu re to direct. 15 H 3 y y N s 16 H 3 y N N y Th is question can be answered knowing the power source of the s i solation valves and that the valves should function at 1 42 F , RO knowledge. Ma ybe we could ask " Under these conditions sho uld an emergence event FU1be declared." As the seco n d part o f the quest i on. RESOLUTION
RESOLUTION: Added time = 5 minutes. And justified question based on operator training.
: Si te was able to justify the question as SRO only based on the N RC guid e. 17 L 2 y y N s 18 H 3 y y N s 19 H 4 y y B s 20 H 4 y y N s 21 H 3 y y M s 22 H 3 y y B s 23 L 2 y y N s 24 H 2 x y y B u Why would anyone bel ieve recording a potential LCO would preven t a re l ease? Answers " C & D" RESOLUTION
58     H     3                                                             y       M   s 59     H     3                                                             y       B   s 60     H     3   x                                                       y       B   l:J For the conditions of the stem I do not believe you can "maintain steady states power and balance loop flows" to maintain steady E
: the question was rewritten. 25 L 3 y y N s}}
states power for the stem conditions it will require an imbalance in loop flow.
RESOLUTION: The conditions of the stem did not require a flow change to comply with Tech Specs, therefore only editorial changes needed to be made 61   L     3                                                             y       B   s
: 1. 2.       3. Psychometric Flaws          4. Job Content Flaws     5. Other 6. 7.                                   8.
Q# LOK  LOO (F/H) (1 -5) Stem Cues T/F     Cred. Partial Job- Minutia #I     Back- Q= SRO B/M/N U/E/S                             Explanation Focus             Dist.         Link         units ward   KIA Only 62   L     4                                                                 y       B   s   Comment: add an explanation for "D" being incorrect to the answer key.
RESOLUTION: Done 63   H     3                                                               y       B   s 64   L     4                                                               y       B   s 65   H     4                                                               y       B   s 66   H     3                                                               y       N   s 67     L     3                                                               y       N   s 68     L     3                                                               y       B   s 69     L     2                                                               y       N   s 70     L     3                                                               y       B   s 71     L     2                                                               y       B   s 72     H     3                                                               y       N   s 73     H     3     x                                                         y       B   y   For a rod drop as described in the stem there would be a ROD DRIFT Alarm. Therefore "A" would be wrong for the same reason as s  MC''.
RESOLUTION: Site explained why there would be no ROD DRIFT Alarm.
74   H     4                             x                                 y       N   y   CPS 401 0.01 step 4.2. 4. Specifically call out Condenser Vacuum Pumps as needing to be considered for rad conditions. Therefore "C" E
is correct.
Editorial: change Stem to. Under these conditions, when 12erforming the SUBSEQUENT ACTIONs OF CPS 4100.01, what special precaution must be taken to minimize the radiological impact of this event when ...
RESOLUTION: Site explained why "C" was incorrect. Only editorial changes were needed.
75   L     4                                                               y       B   s
: 1. 2.       3. Psychometric Flaws         4. Job Content Flaws     5. Other 6. 7.                                     8.
Q# LOK   LOO (F/H) (1 -5) Stem Cues T/F     Cred. Partial Job- Minutia #I     Back- Q = SRO B/M/N U/E/S                                 Explanation Focus             Dist.         Link         units wa rd  KIA Only s   R     0             0   N     L     y             Q     u     E   s     T I     0   N   s 1   H     3                                                               y     y N     s 2   H     3                                                               y     y N     s 3   H     3                                                               y     y M     s 4   H     3                                                               y     y M     s 5   H     3                                                               y     y N     s 6   H     3     x                                                         y     y N     y         1}   The question "If the appropriate actions are taken, can a plant startup commence?" is too vague you can always E
start up after all appropriate action are taken. Did you mean " Does Tech Spec 3.3.1.1 allow restart without repair of the failed channels?"
: 2)   Could CH A and D have tripped at 19:42: 12:15.!1,? If so cou ld this confuse the candidates.
RESOLUTION: Only editorial changes were needed in this question 7   H     3                                                               y     N N     y   This question is written at RO level. i.e. With current conditions should feedwater be shutoff, should HPCI be shutoff, should MSIVs be closed.
RESOLUTION: Site was able to justify the question as SRO only based on the NRC guide.
8   H     2                                                               y     y N     s 9   H     3                                                               y     N B     y   This question does not include
* selection of procedures" only RO level evaluation of plant condition. Can you change to "enter CPS E
XXX to spray the containment, etc" RESOLUTION: Only editorial changes where needed.
10   H     3                                                               y     N B     s 11   H     3                                                               y     y N     s
: 1. 2.       3. Psychometric Flaws          4 . Job Content Flaws   5. Other 6. 7.                                       8.
Q# LOK   LOO (F/H) (1-5) Stem Cues T/F     Cred. Partial Job- Minutia #I Back-     Q= SRO B/M/N U/E/S                               Explanation Focus              Dist.         Link          units ward KIA Only 12   H     4                                                               y     y N   s 13   H     3                                                               y     y N   s 14   H     3                                                               y     y N   s   Note: this was accepted as SRO because the candidate was effectively asked to select a section of the SBO procedure to direct.
15   H     3                                                               y     y N   s 16   H     3                                                               y     N N   y   This question can be answered knowing the power source of the isolation valves and that the valves should function at 142 F, RO knowledge. Maybe we could ask "Under these conditions should an emergence event FU1be declared." As the second part of the question.
RESOLUTION: Site was able to justify the question as SRO only based on the NRC guide.
17   L     2                                                               y     y N   s 18   H     3                                                               y     y N   s 19   H     4                                                               y     y B   s 20   H     4                                                               y     y N   s 21   H     3                                                               y     y M   s 22   H     3                                                               y     y B   s 23   L   2                                                               y     y N   s 24   H     2                       x                                     y     y B   u   Why would anyone believe recording a potential LCO would prevent a release? Answers "C & D" RESOLUTION: the question was rewritten .
25   L   3                                                               y     y N   s
 
ES-401                                     Written Examination Review Worksheet CLINTON 2015                                                         Form ES-401-9
: 1. 2.       3. Psychometric Flaws         4 . Job Content Flaws     5. Other   6. 7.                                     8.
Q# LOK   LOO (F/H) (1-5) Stem Cues     T/F   Cred. Partial Job- Minutia #I       Back- Q=   SRO B/M/N U/E/S                               Explanation Focus               Dist.       Link           units ward   KIA   Only 1   L     3     x                         x                                 y         N     u   The stem states "IRM detectors are withdrawn from the core region during power operation within a               tube which extends from the core plate up to the top guide." This question has two problems
: 1) the IRMs run in dry tubes that extend from the below the reactor vessel to the top guide. So there is no correct answer.
: 2) the dry tub is inserted into a guide tub . So both the "IRMs are within a dry tube" and "the IRMs are with in a guide tube" would be correct.
RESOLUTION: Question replaced 2   H     3                                                                 y         B     s 3   L     3                                                                 y         B     E   The stem now reads ''Which ONE of the following will generate a reactor scram SIGNAL as a result of this loss of power?" Consider changing "SIGNAL* to "input signal". The stem conditions would only be a 'h scram?
RESOLUTION: Change made
 
Instructions (Refer to Section D of ES-401 and Appendix B for additional information regarding each of the following concepts.)
: 1. Enter the level of knowledge (LOK) of each question as either (F)undamental or (H)igher cognitive level.
: 2. Enter the level of difficulty (LOO) of each question using a 1 - 5 (easy - difficult) rating scale (questions in the 2 - 4 range are acceptable).
: 3. Check the appropriate box if a psychometric flaw is identified:
The stem lacks sufficient focus to elicit the correct answer (e.g ., unclear intent, more information is needed, or too much needless information).
The stem or distractors contain cues (i.e., clues. specific determiners, phrasing, length, etc).
The answer choices are a collection of unrelated true/false statements.
The distractors are not credible; single implausible distractors should be repaired , more than one is unacceptable.
One or more distractors is (are) partially correct (e.g., if the applicant can make unstated assumptions that are not contradicted by stem).
: 4. Check the appropriate box if a job content error is identified:
The question is not linked to the job requirements (i.e., the question has a valid KIA but, as written, is not operational in content).
The question requires the recall of knowledge that is too specific for the closed reference test mode (i.e., it is not required to be known from memory).
The question contains data with an unrealistic level of accuracy or inconsistent units (e.g., panel meter in percent with question in gallons).
The question requires reverse logic or application compared to the job requirements.
: 5. Check questions that are sampled fo r conformance with the approved KIA and those that are designated SRO-only (KIA and license level mismatches are unacceptable).
: 6. Enter question source: (B)ank, (M)odified, or (N)ew. Check that (M)odified questions meet criteria of ES-401 Section 0 .2.f.
: 7. Based on the reviewer's judgment, is the question as written (U)nsatisfactory (requiring repair or replacement) , in need of (E)ditorial enhancement, or (S)atisfactory?
: a. At a minimum, explain any "U" ratings (e.g., how the Appendix B psychometric attributes are not being met).
 
ES-401                                                                           2                                                                       Form ES-401-9
: 1. 2.       3. Psychometric Flaws         4 . Job Content Flaws     5. Other   6. 7.                                         8.
Q#   LOK LOO (FIH) (1-5) Stem Cues   T/F   Cred. Partial Job- Minutia #I       Back- Q=   SRO B/M/N U/E/S                                 Explanation Focus               Dist         Link           units ward   KIA Only 4   H     3                                                                 y         B     s 5   H     4                                                                 y         B     s 6   H     4                                                                 y         N     s 7   H     2                                                                 y         N     s 8   L     4                                                                 y         B     s 9     L   3                                                                 y         N     s 10   H     4                                                                 y         8     s 11   H     3                                                                 y         N     s 12   H     3                                                                 y         B     s 13   H     2                                                                 y         N     u   This questions has 2 problems:
: 1) If in the condition of the stern the Governor is held in the raise position long enough to see a rise in both watts and current I believe you would also see a rise in vars. Then there wou ld be no correct answer. (on the simu lator try raising load from 3000 to 3500 Kw and see if vars change .)
: 2) JPM " Parallel DG 18 With Offsite Power
* will duplicate th is question.
RESOLUTION: Question revised 14    H     2                                                                 y         N     s 15     H     3                                                               y         B     s 16     H   4                                                                 y         N     s 17     L     3                                                               y         B     s 18     H     3                                                               y         N     s
: 1. 2.       3. PsychOmetric Flaws           4. Job Content Flaws   5. Other 6. 7.                                         8.
Q# LOK  LOO (FIH) (1-5) Stem Cues   T/F   Cred   Partial Job- Minutia #I Back-   Q= SRO B/M/N U/E/S                                   Explanation Focus             Dist.         Link         units ward KJA Only 19    H     3                                                               y       B   s 20   H     3                                                               y       B   s 21   H     3     x                                                         y       N   IJ I do not think this is an RO level TS question as written. Consider changing the bullets to E
                                                                                                    -     1 E21-F011 LPCS Pump Min Flow Recirc Valve be closed with fuses removed .
                                                                                                    -     AUTO start of the LPCS disabled .
RESOLUTION: Site supported that the question met the expectations of ROs at Clinton . Editorial changes made.
22   H     3                                                               y       B   s 23   H     2                                                               y       N   s 24    L   3                               x                               y       B   u   Between LPRM calibrations APRM gains would be adjusted to account for LPRM detector U235 depletion.
RESOLUTION: The second correct answer was changed.
25   H     3                                                             y       B   s 26     L     3                                                             y       N   s 27     L     3                             x                               y       M   u   If "D" is correct then "a** will be correct. Consider changing "A" and "B" (1) to ' primary but NOT secondary" .
RESOLUTION: Requested change made 28     l     3                                                             y       B   s 29     l     3                                                             y       B   s 30     L     3                                                             y       B   s 31     H     3                                                             y       N   s 32     L     3                                                             y       N   s 33   L     4                                                             y       N   s 34   H     3                                                             y       N   s
: 1. 2.       3. Psychometric Flaws         4. Job Content Flaws     5. Other  6. 7.                                         8.
Q# LOK  LOO (F/H) (1-5) Stem Cues T/F     Cred. Partial Job- Minutia #I     Back- Q=   SRO B/M/N U/E/S                                 Explanation Focus             Dist.         Link         units ward   KIA  Only 35   H     2                                                               y         B     s 36   L     3                                                               y         N     s 37   l     3                             x                                 y         B     y   "B" p revent boron dilution,   *c* prevent injection inside the shroud are both correct. See EOP-1A reference provided.
E RESOLUTION: The stem was changed to clarify the quest ion.
38   L     2                                                               y         N     s 39   H     3                                                               y         N     s 40   L     3                                                               y         6     s 41   l     3                                                               y         B     s 42   H     3                                                               y         M     s 43   H     3                                                               y         B     s 44   l     3                                                               y         N     s 45    L   2                                                               y         B     E   List the distractors lowest to highest. i.e. switch "B' and "C".
RESOLUTION: Change made 46  H     3                                                               y         N     s 47   H     3                                                               y         N     s 48   L   3                                                               y         B     s 49  H     3                       x                                       y         B     E         1)   For the conditions given in the stem is it expected that the cool down rate will increase or decrease with time? *s* will be correct only with an increasing or constant cool down rate.
: 2)   For distracte r "A ' should it be changed to "Shift RCIR discharge to the reactor vessel"? Because with RCIR in tank to tank vessel level would be unchanged by raising flow.
RESOLUTION: Stem and distracter changed 50   H     2                                                               y         B     s
: 1. 2.       3. Psychometric Flaws         4. Job Content Flaws   5. Other 6. 7.                                         8.
Q#   LOK LOO (F/H) (1-5) Stem Cues T/F     Cred. Partial Job- Minutia #I Back-   Q= SRO B/M/N U/E/S                                 Explanation Focus             Dist.         Link         units ward KIA Only 51   L     3     x                                                       y       N   u   It is not clear in the stem if in part (2) of the question we are asking for the action to be taken at the time level is 18ft 9inches (answer D),
or at the 15ft 1 inch (Answer C)
RESOLUTION: Question revised.
52   L     3                                                             y       B   E   Change "B" to, "lowest possible driving head and flow from the primary system!:tO reduce the discharge rate."
RESOLUTION: Change made 53   L     3                                                             y       B   s 54   H     3                                                             y       M   s 55     L     3                                                             y       N   s 56   H     3                                                             y       N   s 57   H     4                                                             y       N   E   I th ink we need to provide the candidates with a time after the loss of offsite power that the plant conditions are current for. (the fact that an SRV is open after a LOFP doesn't= ATWS at time 0) Also do you think it is fair to ask an operator to diagnose an ATWS and not give them control rod or APRM info. Maybe we should say you are the BOP and observe the conditions.
RESOLUTION: Added time = 5 minutes. And justified question based on operator training.
58     H     3                                                             y       M   s 59     H     3                                                             y       B   s 60     H     3   x                                                       y       B   l:J For the conditions of the stem I do not believe you can "maintain steady states power and balance loop flows" to maintain steady E
states power for the stem conditions it will require an imbalance in loop flow.
RESOLUTION: The conditions of the stem did not require a flow change to comply with Tech Specs, therefore only editorial changes needed to be made 61   L     3                                                             y       B   s
: 1. 2.       3. Psychometric Flaws          4. Job Content Flaws     5. Other 6. 7.                                   8.
Q# LOK  LOO (F/H) (1 -5) Stem Cues T/F     Cred. Partial Job- Minutia #I     Back- Q= SRO B/M/N U/E/S                             Explanation Focus             Dist.         Link         units ward   KIA Only 62   L     4                                                                 y       B   s   Comment: add an explanation for "D" being incorrect to the answer key.
RESOLUTION: Done 63   H     3                                                               y       B   s 64   L     4                                                               y       B   s 65   H     4                                                               y       B   s 66   H     3                                                               y       N   s 67     L     3                                                               y       N   s 68     L     3                                                               y       B   s 69     L     2                                                               y       N   s 70     L     3                                                               y       B   s 71     L     2                                                               y       B   s 72     H     3                                                               y       N   s 73     H     3     x                                                         y       B   y   For a rod drop as described in the stem there would be a ROD DRIFT Alarm. Therefore "A" would be wrong for the same reason as s  MC''.
RESOLUTION: Site explained why there would be no ROD DRIFT Alarm.
74   H     4                             x                                 y       N   y   CPS 401 0.01 step 4.2. 4. Specifically call out Condenser Vacuum Pumps as needing to be considered for rad conditions. Therefore "C" E
is correct.
Editorial: change Stem to. Under these conditions, when 12erforming the SUBSEQUENT ACTIONs OF CPS 4100.01, what special precaution must be taken to minimize the radiological impact of this event when ...
RESOLUTION: Site explained why "C" was incorrect. Only editorial changes were needed.
75   L     4                                                               y       B   s
: 1. 2.       3. Psychometric Flaws         4. Job Content Flaws     5. Other 6. 7.                                     8.
Q# LOK   LOO (F/H) (1 -5) Stem Cues T/F     Cred. Partial Job- Minutia #I     Back- Q = SRO B/M/N U/E/S                                 Explanation Focus             Dist.         Link         units wa rd  KIA Only s   R     0             0   N     L     y             Q     u     E   s     T I     0   N   s 1   H     3                                                               y     y N     s 2   H     3                                                               y     y N     s 3   H     3                                                               y     y M     s 4   H     3                                                               y     y M     s 5   H     3                                                               y     y N     s 6   H     3     x                                                         y     y N     y         1}   The question "If the appropriate actions are taken, can a plant startup commence?" is too vague you can always E
start up after all appropriate action are taken. Did you mean " Does Tech Spec 3.3.1.1 allow restart without repair of the failed channels?"
: 2)   Could CH A and D have tripped at 19:42: 12:15.!1,? If so cou ld this confuse the candidates.
RESOLUTION: Only editorial changes were needed in this question 7   H     3                                                               y     N N     y   This question is written at RO level. i.e. With current conditions should feedwater be shutoff, should HPCI be shutoff, should MSIVs be closed.
RESOLUTION: Site was able to justify the question as SRO only based on the NRC guide.
8   H     2                                                               y     y N     s 9   H     3                                                               y     N B     y   This question does not include
* selection of procedures" only RO level evaluation of plant condition. Can you change to "enter CPS E
XXX to spray the containment, etc" RESOLUTION: Only editorial changes where needed.
10   H     3                                                               y     N B     s 11   H     3                                                               y     y N     s
: 1. 2.       3. Psychometric Flaws          4 . Job Content Flaws   5. Other 6. 7.                                       8.
Q# LOK   LOO (F/H) (1-5) Stem Cues T/F     Cred. Partial Job- Minutia #I Back-     Q= SRO B/M/N U/E/S                               Explanation Focus              Dist.         Link          units ward KIA Only 12   H     4                                                               y     y N   s 13   H     3                                                               y     y N   s 14   H     3                                                               y     y N   s   Note: this was accepted as SRO because the candidate was effectively asked to select a section of the SBO procedure to direct.
15   H     3                                                               y     y N   s 16   H     3                                                               y     N N   y   This question can be answered knowing the power source of the isolation valves and that the valves should function at 142 F, RO knowledge. Maybe we could ask "Under these conditions should an emergence event FU1be declared." As the second part of the question.
RESOLUTION: Site was able to justify the question as SRO only based on the NRC guide.
17   L     2                                                               y     y N   s 18   H     3                                                               y     y N   s 19   H     4                                                               y     y B   s 20   H     4                                                               y     y N   s 21   H     3                                                               y     y M   s 22   H     3                                                               y     y B   s 23   L   2                                                               y     y N   s 24   H     2                       x                                     y     y B   u   Why would anyone believe recording a potential LCO would prevent a release? Answers "C & D" RESOLUTION: the question was rewritten .
25   L   3                                                               y     y N   s}}

Latest revision as of 05:50, 31 October 2019

2015 Clinton Power Station Initial License Examination Form ES-401-9
ML15313A063
Person / Time
Site: Clinton Constellation icon.png
Issue date: 11/06/2015
From:
Division of Reactor Safety III
To:
Zoia, C D
Shared Package
ML15124A054 List:
References
Download: ML15313A063 (9)


Text

ES-401 Written Examination Review Worksheet CLINTON 2015 Form ES-401-9

1. 2. 3. Psychometric Flaws 4 . Job Content Flaws 5. Other 6. 7. 8.

Q# LOK LOO (F/H) (1-5) Stem Cues T/F Cred. Partial Job- Minutia #I Back- Q= SRO B/M/N U/E/S Explanation Focus Dist. Link units ward KIA Only 1 L 3 x x y N u The stem states "IRM detectors are withdrawn from the core region during power operation within a tube which extends from the core plate up to the top guide." This question has two problems

1) the IRMs run in dry tubes that extend from the below the reactor vessel to the top guide. So there is no correct answer.
2) the dry tub is inserted into a guide tub . So both the "IRMs are within a dry tube" and "the IRMs are with in a guide tube" would be correct.

RESOLUTION: Question replaced 2 H 3 y B s 3 L 3 y B E The stem now reads Which ONE of the following will generate a reactor scram SIGNAL as a result of this loss of power?" Consider changing "SIGNAL* to "input signal". The stem conditions would only be a 'h scram?

RESOLUTION: Change made

Instructions (Refer to Section D of ES-401 and Appendix B for additional information regarding each of the following concepts.)

1. Enter the level of knowledge (LOK) of each question as either (F)undamental or (H)igher cognitive level.
2. Enter the level of difficulty (LOO) of each question using a 1 - 5 (easy - difficult) rating scale (questions in the 2 - 4 range are acceptable).
3. Check the appropriate box if a psychometric flaw is identified:

The stem lacks sufficient focus to elicit the correct answer (e.g ., unclear intent, more information is needed, or too much needless information).

The stem or distractors contain cues (i.e., clues. specific determiners, phrasing, length, etc).

The answer choices are a collection of unrelated true/false statements.

The distractors are not credible; single implausible distractors should be repaired , more than one is unacceptable.

One or more distractors is (are) partially correct (e.g., if the applicant can make unstated assumptions that are not contradicted by stem).

4. Check the appropriate box if a job content error is identified:

The question is not linked to the job requirements (i.e., the question has a valid KIA but, as written, is not operational in content).

The question requires the recall of knowledge that is too specific for the closed reference test mode (i.e., it is not required to be known from memory).

The question contains data with an unrealistic level of accuracy or inconsistent units (e.g., panel meter in percent with question in gallons).

The question requires reverse logic or application compared to the job requirements.

5. Check questions that are sampled fo r conformance with the approved KIA and those that are designated SRO-only (KIA and license level mismatches are unacceptable).
6. Enter question source: (B)ank, (M)odified, or (N)ew. Check that (M)odified questions meet criteria of ES-401 Section 0 .2.f.
7. Based on the reviewer's judgment, is the question as written (U)nsatisfactory (requiring repair or replacement) , in need of (E)ditorial enhancement, or (S)atisfactory?
a. At a minimum, explain any "U" ratings (e.g., how the Appendix B psychometric attributes are not being met).

ES-401 2 Form ES-401-9

1. 2. 3. Psychometric Flaws 4 . Job Content Flaws 5. Other 6. 7. 8.

Q# LOK LOO (FIH) (1-5) Stem Cues T/F Cred. Partial Job- Minutia #I Back- Q= SRO B/M/N U/E/S Explanation Focus Dist Link units ward KIA Only 4 H 3 y B s 5 H 4 y B s 6 H 4 y N s 7 H 2 y N s 8 L 4 y B s 9 L 3 y N s 10 H 4 y 8 s 11 H 3 y N s 12 H 3 y B s 13 H 2 y N u This questions has 2 problems:

1) If in the condition of the stern the Governor is held in the raise position long enough to see a rise in both watts and current I believe you would also see a rise in vars. Then there wou ld be no correct answer. (on the simu lator try raising load from 3000 to 3500 Kw and see if vars change .)
2) JPM " Parallel DG 18 With Offsite Power
  • will duplicate th is question.

RESOLUTION: Question revised 14 H 2 y N s 15 H 3 y B s 16 H 4 y N s 17 L 3 y B s 18 H 3 y N s

1. 2. 3. PsychOmetric Flaws 4. Job Content Flaws 5. Other 6. 7. 8.

Q# LOK LOO (FIH) (1-5) Stem Cues T/F Cred Partial Job- Minutia #I Back- Q= SRO B/M/N U/E/S Explanation Focus Dist. Link units ward KJA Only 19 H 3 y B s 20 H 3 y B s 21 H 3 x y N IJ I do not think this is an RO level TS question as written. Consider changing the bullets to E

- 1 E21-F011 LPCS Pump Min Flow Recirc Valve be closed with fuses removed .

- AUTO start of the LPCS disabled .

RESOLUTION: Site supported that the question met the expectations of ROs at Clinton . Editorial changes made.

22 H 3 y B s 23 H 2 y N s 24 L 3 x y B u Between LPRM calibrations APRM gains would be adjusted to account for LPRM detector U235 depletion.

RESOLUTION: The second correct answer was changed.

25 H 3 y B s 26 L 3 y N s 27 L 3 x y M u If "D" is correct then "a** will be correct. Consider changing "A" and "B" (1) to ' primary but NOT secondary" .

RESOLUTION: Requested change made 28 l 3 y B s 29 l 3 y B s 30 L 3 y B s 31 H 3 y N s 32 L 3 y N s 33 L 4 y N s 34 H 3 y N s

1. 2. 3. Psychometric Flaws 4. Job Content Flaws 5. Other 6. 7. 8.

Q# LOK LOO (F/H) (1-5) Stem Cues T/F Cred. Partial Job- Minutia #I Back- Q= SRO B/M/N U/E/S Explanation Focus Dist. Link units ward KIA Only 35 H 2 y B s 36 L 3 y N s 37 l 3 x y B y "B" p revent boron dilution, *c* prevent injection inside the shroud are both correct. See EOP-1A reference provided.

E RESOLUTION: The stem was changed to clarify the quest ion.

38 L 2 y N s 39 H 3 y N s 40 L 3 y 6 s 41 l 3 y B s 42 H 3 y M s 43 H 3 y B s 44 l 3 y N s 45 L 2 y B E List the distractors lowest to highest. i.e. switch "B' and "C".

RESOLUTION: Change made 46 H 3 y N s 47 H 3 y N s 48 L 3 y B s 49 H 3 x y B E 1) For the conditions given in the stem is it expected that the cool down rate will increase or decrease with time? *s* will be correct only with an increasing or constant cool down rate.

2) For distracte r "A ' should it be changed to "Shift RCIR discharge to the reactor vessel"? Because with RCIR in tank to tank vessel level would be unchanged by raising flow.

RESOLUTION: Stem and distracter changed 50 H 2 y B s

1. 2. 3. Psychometric Flaws 4. Job Content Flaws 5. Other 6. 7. 8.

Q# LOK LOO (F/H) (1-5) Stem Cues T/F Cred. Partial Job- Minutia #I Back- Q= SRO B/M/N U/E/S Explanation Focus Dist. Link units ward KIA Only 51 L 3 x y N u It is not clear in the stem if in part (2) of the question we are asking for the action to be taken at the time level is 18ft 9inches (answer D),

or at the 15ft 1 inch (Answer C)

RESOLUTION: Question revised.

52 L 3 y B E Change "B" to, "lowest possible driving head and flow from the primary system!:tO reduce the discharge rate."

RESOLUTION: Change made 53 L 3 y B s 54 H 3 y M s 55 L 3 y N s 56 H 3 y N s 57 H 4 y N E I th ink we need to provide the candidates with a time after the loss of offsite power that the plant conditions are current for. (the fact that an SRV is open after a LOFP doesn't= ATWS at time 0) Also do you think it is fair to ask an operator to diagnose an ATWS and not give them control rod or APRM info. Maybe we should say you are the BOP and observe the conditions.

RESOLUTION: Added time = 5 minutes. And justified question based on operator training.

58 H 3 y M s 59 H 3 y B s 60 H 3 x y B l:J For the conditions of the stem I do not believe you can "maintain steady states power and balance loop flows" to maintain steady E

states power for the stem conditions it will require an imbalance in loop flow.

RESOLUTION: The conditions of the stem did not require a flow change to comply with Tech Specs, therefore only editorial changes needed to be made 61 L 3 y B s

1. 2. 3. Psychometric Flaws 4. Job Content Flaws 5. Other 6. 7. 8.

Q# LOK LOO (F/H) (1 -5) Stem Cues T/F Cred. Partial Job- Minutia #I Back- Q= SRO B/M/N U/E/S Explanation Focus Dist. Link units ward KIA Only 62 L 4 y B s Comment: add an explanation for "D" being incorrect to the answer key.

RESOLUTION: Done 63 H 3 y B s 64 L 4 y B s 65 H 4 y B s 66 H 3 y N s 67 L 3 y N s 68 L 3 y B s 69 L 2 y N s 70 L 3 y B s 71 L 2 y B s 72 H 3 y N s 73 H 3 x y B y For a rod drop as described in the stem there would be a ROD DRIFT Alarm. Therefore "A" would be wrong for the same reason as s MC.

RESOLUTION: Site explained why there would be no ROD DRIFT Alarm.

74 H 4 x y N y CPS 401 0.01 step 4.2. 4. Specifically call out Condenser Vacuum Pumps as needing to be considered for rad conditions. Therefore "C" E

is correct.

Editorial: change Stem to. Under these conditions, when 12erforming the SUBSEQUENT ACTIONs OF CPS 4100.01, what special precaution must be taken to minimize the radiological impact of this event when ...

RESOLUTION: Site explained why "C" was incorrect. Only editorial changes were needed.

75 L 4 y B s

1. 2. 3. Psychometric Flaws 4. Job Content Flaws 5. Other 6. 7. 8.

Q# LOK LOO (F/H) (1 -5) Stem Cues T/F Cred. Partial Job- Minutia #I Back- Q = SRO B/M/N U/E/S Explanation Focus Dist. Link units wa rd KIA Only s R 0 0 N L y Q u E s T I 0 N s 1 H 3 y y N s 2 H 3 y y N s 3 H 3 y y M s 4 H 3 y y M s 5 H 3 y y N s 6 H 3 x y y N y 1} The question "If the appropriate actions are taken, can a plant startup commence?" is too vague you can always E

start up after all appropriate action are taken. Did you mean " Does Tech Spec 3.3.1.1 allow restart without repair of the failed channels?"

2) Could CH A and D have tripped at 19:42: 12:15.!1,? If so cou ld this confuse the candidates.

RESOLUTION: Only editorial changes were needed in this question 7 H 3 y N N y This question is written at RO level. i.e. With current conditions should feedwater be shutoff, should HPCI be shutoff, should MSIVs s be closed.

RESOLUTION: Site was able to justify the question as SRO only based on the NRC guide.

8 H 2 y y N s 9 H 3 y N B y This question does not include

  • selection of procedures" only RO level evaluation of plant condition. Can you change to "enter CPS E

XXX to spray the containment, etc" RESOLUTION: Only editorial changes where needed.

10 H 3 y N B s 11 H 3 y y N s

1. 2. 3. Psychometric Flaws 4 . Job Content Flaws 5. Other 6. 7. 8.

Q# LOK LOO (F/H) (1-5) Stem Cues T/F Cred. Partial Job- Minutia #I Back- Q= SRO B/M/N U/E/S Explanation Focus Dist. Link units ward KIA Only 12 H 4 y y N s 13 H 3 y y N s 14 H 3 y y N s Note: this was accepted as SRO because the candidate was effectively asked to select a section of the SBO procedure to direct.

15 H 3 y y N s 16 H 3 y N N y This question can be answered knowing the power source of the isolation valves and that the valves should function at 142 F, RO s knowledge. Maybe we could ask "Under these conditions should an emergence event FU1be declared." As the second part of the question.

RESOLUTION: Site was able to justify the question as SRO only based on the NRC guide.

17 L 2 y y N s 18 H 3 y y N s 19 H 4 y y B s 20 H 4 y y N s 21 H 3 y y M s 22 H 3 y y B s 23 L 2 y y N s 24 H 2 x y y B u Why would anyone believe recording a potential LCO would prevent a release? Answers "C & D" RESOLUTION: the question was rewritten .

25 L 3 y y N s

ES-401 Written Examination Review Worksheet CLINTON 2015 Form ES-401-9

1. 2. 3. Psychometric Flaws 4 . Job Content Flaws 5. Other 6. 7. 8.

Q# LOK LOO (F/H) (1-5) Stem Cues T/F Cred. Partial Job- Minutia #I Back- Q= SRO B/M/N U/E/S Explanation Focus Dist. Link units ward KIA Only 1 L 3 x x y N u The stem states "IRM detectors are withdrawn from the core region during power operation within a tube which extends from the core plate up to the top guide." This question has two problems

1) the IRMs run in dry tubes that extend from the below the reactor vessel to the top guide. So there is no correct answer.
2) the dry tub is inserted into a guide tub . So both the "IRMs are within a dry tube" and "the IRMs are with in a guide tube" would be correct.

RESOLUTION: Question replaced 2 H 3 y B s 3 L 3 y B E The stem now reads Which ONE of the following will generate a reactor scram SIGNAL as a result of this loss of power?" Consider changing "SIGNAL* to "input signal". The stem conditions would only be a 'h scram?

RESOLUTION: Change made

Instructions (Refer to Section D of ES-401 and Appendix B for additional information regarding each of the following concepts.)

1. Enter the level of knowledge (LOK) of each question as either (F)undamental or (H)igher cognitive level.
2. Enter the level of difficulty (LOO) of each question using a 1 - 5 (easy - difficult) rating scale (questions in the 2 - 4 range are acceptable).
3. Check the appropriate box if a psychometric flaw is identified:

The stem lacks sufficient focus to elicit the correct answer (e.g ., unclear intent, more information is needed, or too much needless information).

The stem or distractors contain cues (i.e., clues. specific determiners, phrasing, length, etc).

The answer choices are a collection of unrelated true/false statements.

The distractors are not credible; single implausible distractors should be repaired , more than one is unacceptable.

One or more distractors is (are) partially correct (e.g., if the applicant can make unstated assumptions that are not contradicted by stem).

4. Check the appropriate box if a job content error is identified:

The question is not linked to the job requirements (i.e., the question has a valid KIA but, as written, is not operational in content).

The question requires the recall of knowledge that is too specific for the closed reference test mode (i.e., it is not required to be known from memory).

The question contains data with an unrealistic level of accuracy or inconsistent units (e.g., panel meter in percent with question in gallons).

The question requires reverse logic or application compared to the job requirements.

5. Check questions that are sampled fo r conformance with the approved KIA and those that are designated SRO-only (KIA and license level mismatches are unacceptable).
6. Enter question source: (B)ank, (M)odified, or (N)ew. Check that (M)odified questions meet criteria of ES-401 Section 0 .2.f.
7. Based on the reviewer's judgment, is the question as written (U)nsatisfactory (requiring repair or replacement) , in need of (E)ditorial enhancement, or (S)atisfactory?
a. At a minimum, explain any "U" ratings (e.g., how the Appendix B psychometric attributes are not being met).

ES-401 2 Form ES-401-9

1. 2. 3. Psychometric Flaws 4 . Job Content Flaws 5. Other 6. 7. 8.

Q# LOK LOO (FIH) (1-5) Stem Cues T/F Cred. Partial Job- Minutia #I Back- Q= SRO B/M/N U/E/S Explanation Focus Dist Link units ward KIA Only 4 H 3 y B s 5 H 4 y B s 6 H 4 y N s 7 H 2 y N s 8 L 4 y B s 9 L 3 y N s 10 H 4 y 8 s 11 H 3 y N s 12 H 3 y B s 13 H 2 y N u This questions has 2 problems:

1) If in the condition of the stern the Governor is held in the raise position long enough to see a rise in both watts and current I believe you would also see a rise in vars. Then there wou ld be no correct answer. (on the simu lator try raising load from 3000 to 3500 Kw and see if vars change .)
2) JPM " Parallel DG 18 With Offsite Power
  • will duplicate th is question.

RESOLUTION: Question revised 14 H 2 y N s 15 H 3 y B s 16 H 4 y N s 17 L 3 y B s 18 H 3 y N s

1. 2. 3. PsychOmetric Flaws 4. Job Content Flaws 5. Other 6. 7. 8.

Q# LOK LOO (FIH) (1-5) Stem Cues T/F Cred Partial Job- Minutia #I Back- Q= SRO B/M/N U/E/S Explanation Focus Dist. Link units ward KJA Only 19 H 3 y B s 20 H 3 y B s 21 H 3 x y N IJ I do not think this is an RO level TS question as written. Consider changing the bullets to E

- 1 E21-F011 LPCS Pump Min Flow Recirc Valve be closed with fuses removed .

- AUTO start of the LPCS disabled .

RESOLUTION: Site supported that the question met the expectations of ROs at Clinton . Editorial changes made.

22 H 3 y B s 23 H 2 y N s 24 L 3 x y B u Between LPRM calibrations APRM gains would be adjusted to account for LPRM detector U235 depletion.

RESOLUTION: The second correct answer was changed.

25 H 3 y B s 26 L 3 y N s 27 L 3 x y M u If "D" is correct then "a** will be correct. Consider changing "A" and "B" (1) to ' primary but NOT secondary" .

RESOLUTION: Requested change made 28 l 3 y B s 29 l 3 y B s 30 L 3 y B s 31 H 3 y N s 32 L 3 y N s 33 L 4 y N s 34 H 3 y N s

1. 2. 3. Psychometric Flaws 4. Job Content Flaws 5. Other 6. 7. 8.

Q# LOK LOO (F/H) (1-5) Stem Cues T/F Cred. Partial Job- Minutia #I Back- Q= SRO B/M/N U/E/S Explanation Focus Dist. Link units ward KIA Only 35 H 2 y B s 36 L 3 y N s 37 l 3 x y B y "B" p revent boron dilution, *c* prevent injection inside the shroud are both correct. See EOP-1A reference provided.

E RESOLUTION: The stem was changed to clarify the quest ion.

38 L 2 y N s 39 H 3 y N s 40 L 3 y 6 s 41 l 3 y B s 42 H 3 y M s 43 H 3 y B s 44 l 3 y N s 45 L 2 y B E List the distractors lowest to highest. i.e. switch "B' and "C".

RESOLUTION: Change made 46 H 3 y N s 47 H 3 y N s 48 L 3 y B s 49 H 3 x y B E 1) For the conditions given in the stem is it expected that the cool down rate will increase or decrease with time? *s* will be correct only with an increasing or constant cool down rate.

2) For distracte r "A ' should it be changed to "Shift RCIR discharge to the reactor vessel"? Because with RCIR in tank to tank vessel level would be unchanged by raising flow.

RESOLUTION: Stem and distracter changed 50 H 2 y B s

1. 2. 3. Psychometric Flaws 4. Job Content Flaws 5. Other 6. 7. 8.

Q# LOK LOO (F/H) (1-5) Stem Cues T/F Cred. Partial Job- Minutia #I Back- Q= SRO B/M/N U/E/S Explanation Focus Dist. Link units ward KIA Only 51 L 3 x y N u It is not clear in the stem if in part (2) of the question we are asking for the action to be taken at the time level is 18ft 9inches (answer D),

or at the 15ft 1 inch (Answer C)

RESOLUTION: Question revised.

52 L 3 y B E Change "B" to, "lowest possible driving head and flow from the primary system!:tO reduce the discharge rate."

RESOLUTION: Change made 53 L 3 y B s 54 H 3 y M s 55 L 3 y N s 56 H 3 y N s 57 H 4 y N E I th ink we need to provide the candidates with a time after the loss of offsite power that the plant conditions are current for. (the fact that an SRV is open after a LOFP doesn't= ATWS at time 0) Also do you think it is fair to ask an operator to diagnose an ATWS and not give them control rod or APRM info. Maybe we should say you are the BOP and observe the conditions.

RESOLUTION: Added time = 5 minutes. And justified question based on operator training.

58 H 3 y M s 59 H 3 y B s 60 H 3 x y B l:J For the conditions of the stem I do not believe you can "maintain steady states power and balance loop flows" to maintain steady E

states power for the stem conditions it will require an imbalance in loop flow.

RESOLUTION: The conditions of the stem did not require a flow change to comply with Tech Specs, therefore only editorial changes needed to be made 61 L 3 y B s

1. 2. 3. Psychometric Flaws 4. Job Content Flaws 5. Other 6. 7. 8.

Q# LOK LOO (F/H) (1 -5) Stem Cues T/F Cred. Partial Job- Minutia #I Back- Q= SRO B/M/N U/E/S Explanation Focus Dist. Link units ward KIA Only 62 L 4 y B s Comment: add an explanation for "D" being incorrect to the answer key.

RESOLUTION: Done 63 H 3 y B s 64 L 4 y B s 65 H 4 y B s 66 H 3 y N s 67 L 3 y N s 68 L 3 y B s 69 L 2 y N s 70 L 3 y B s 71 L 2 y B s 72 H 3 y N s 73 H 3 x y B y For a rod drop as described in the stem there would be a ROD DRIFT Alarm. Therefore "A" would be wrong for the same reason as s MC.

RESOLUTION: Site explained why there would be no ROD DRIFT Alarm.

74 H 4 x y N y CPS 401 0.01 step 4.2. 4. Specifically call out Condenser Vacuum Pumps as needing to be considered for rad conditions. Therefore "C" E

is correct.

Editorial: change Stem to. Under these conditions, when 12erforming the SUBSEQUENT ACTIONs OF CPS 4100.01, what special precaution must be taken to minimize the radiological impact of this event when ...

RESOLUTION: Site explained why "C" was incorrect. Only editorial changes were needed.

75 L 4 y B s

1. 2. 3. Psychometric Flaws 4. Job Content Flaws 5. Other 6. 7. 8.

Q# LOK LOO (F/H) (1 -5) Stem Cues T/F Cred. Partial Job- Minutia #I Back- Q = SRO B/M/N U/E/S Explanation Focus Dist. Link units wa rd KIA Only s R 0 0 N L y Q u E s T I 0 N s 1 H 3 y y N s 2 H 3 y y N s 3 H 3 y y M s 4 H 3 y y M s 5 H 3 y y N s 6 H 3 x y y N y 1} The question "If the appropriate actions are taken, can a plant startup commence?" is too vague you can always E

start up after all appropriate action are taken. Did you mean " Does Tech Spec 3.3.1.1 allow restart without repair of the failed channels?"

2) Could CH A and D have tripped at 19:42: 12:15.!1,? If so cou ld this confuse the candidates.

RESOLUTION: Only editorial changes were needed in this question 7 H 3 y N N y This question is written at RO level. i.e. With current conditions should feedwater be shutoff, should HPCI be shutoff, should MSIVs s be closed.

RESOLUTION: Site was able to justify the question as SRO only based on the NRC guide.

8 H 2 y y N s 9 H 3 y N B y This question does not include

  • selection of procedures" only RO level evaluation of plant condition. Can you change to "enter CPS E

XXX to spray the containment, etc" RESOLUTION: Only editorial changes where needed.

10 H 3 y N B s 11 H 3 y y N s

1. 2. 3. Psychometric Flaws 4 . Job Content Flaws 5. Other 6. 7. 8.

Q# LOK LOO (F/H) (1-5) Stem Cues T/F Cred. Partial Job- Minutia #I Back- Q= SRO B/M/N U/E/S Explanation Focus Dist. Link units ward KIA Only 12 H 4 y y N s 13 H 3 y y N s 14 H 3 y y N s Note: this was accepted as SRO because the candidate was effectively asked to select a section of the SBO procedure to direct.

15 H 3 y y N s 16 H 3 y N N y This question can be answered knowing the power source of the isolation valves and that the valves should function at 142 F, RO s knowledge. Maybe we could ask "Under these conditions should an emergence event FU1be declared." As the second part of the question.

RESOLUTION: Site was able to justify the question as SRO only based on the NRC guide.

17 L 2 y y N s 18 H 3 y y N s 19 H 4 y y B s 20 H 4 y y N s 21 H 3 y y M s 22 H 3 y y B s 23 L 2 y y N s 24 H 2 x y y B u Why would anyone believe recording a potential LCO would prevent a release? Answers "C & D" RESOLUTION: the question was rewritten .

25 L 3 y y N s